Finite Well Problem: Can Energy Be Equal to V0?

  • I
  • Thread starter Coelum
  • Start date
  • Tags
    Finite
In summary, the conversation revolved around the question of whether the energy of a particle in a bound state can be equal to the potential depth of the well. The conclusion was that this is not possible due to the nature of the Schrodinger equation and boundary conditions. The case of setting the energy equal to the potential depth results in no solutions. Theoretical considerations suggest that this would correspond to an unbounded, linearly increasing wave function outside the well. However, this does not correspond to a physically possible state.
  • #1
Coelum
97
32
TL;DR Summary
Do bound solutions exist for E=V0 where V0 is the depth of the well?
Dear PFers,
I have a question about the finite potential well problem. Let's assume the well is centered in 0 and the potential is V0 outside the well and 0 inside. For more details see Wikipedia: https://en.wikipedia.org/wiki/Finite_potential_well.

Now, the question is: can the energy of the particle in a bound state be equal to V0?
I expect it cannot, as:
  1. the wave-function outside the box should satisfy the free particle Schrodinger equation with null energy (essentially, the second derivative should be 0)
  2. the solution of the equation described above is a linear function
  3. a linear function cannot be normalized if the domain is unbound
  4. the domain outside the well is unbound (in fact, we have two, separate, unbound regions on both sides).
Is my reasoning correct?
 
Physics news on Phys.org
  • #2
Coelum said:
TL;DR Summary: Do bound solutions exist for E=V0 where V0 is the depth of the well?

Dear PFers,
I have a question about the finite potential well problem. Let's assume the well is centered in 0 and the potential is V0 outside the well and 0 inside. For more details see Wikipedia: https://en.wikipedia.org/wiki/Finite_potential_well.

Now, the question is: can the energy of the particle in a bound state be equal to V0?
An energy of ##E < V_0## represents a bound state; and ##E > V_0## represents a scattering state. Is your question what happens if we look for ##E = V_0##.

The short answer is that it must be something of an unstable state, where technically it cannot escape to infinity, but not is it in a stable bound state.

There is a finite set of bound states (with ##E < V_0##), but the highest bound energy is always less than ##V_0##. I guess you could play about with the solution to see what parameters allow you to get closest to ##E = V_0##.
Coelum said:
I expect it cannot, as:
  1. the wave-function outside the box should satisfy the free particle Schrodinger equation with null energy (essentially, the second derivative should be 0)
  2. the solution of the equation described above is a linear function
I think you are confusing the energy level (which is just a single eigenvalue) with a constant wavefunction. For ##E = V_0##, the wavefunction would be zero outside the well.
Coelum said:
  1. a linear function cannot be normalized if the domain is unbound
  2. the domain outside the well is unbound (in fact, we have two, separate, unbound regions on both sides).
Is my reasoning correct?
I don't think so.
 
  • #3
Thanks PeroK for your prompt reply.
PeroK said:
I think you are confusing the energy level (which is just a single eigenvalue) with a constant wavefunction. For E=V0, the wavefunction would be zero outside the well.
From your answer, I guess I was not clear. Let me add a few details.

If ## E=V_0 ## the Schrodinger equation outside the well becomes
$$ -\frac{\hbar^2}{2m}\frac{d^2\Psi}{dx^2} = 0 $$
with solution a linear function (## A,B\in \mathcal R ##)
$$ \Psi(x)=Ax+B. $$
Inside the well the S.E. is
$$ -\frac{\hbar^2}{2m}\frac{d^2\Psi}{dx^2} = E\Psi $$
$$ \frac{d^2\Psi}{dx^2} = -\frac{2mE}{\hbar^2}\Psi = -K^2\Psi $$
with solution (## C,D\in \mathcal R ##)
$$ \Psi(x)=C\sin(Kx)+D\cos(Kx). $$
The normalization condition for a single particle imposes ## A=B=0 ## on both sides of the well. The boundary conditions for the continuity of ## \Psi ## and its first derivative at the borders of the well lead to the null solution as the only solution. That's what I meant in my previous statements.

If you still believe my reasoning is wrong, please help me by pointing out the error in the derivation above (including computing the boundary conditions, whose boring details I shamelessly skipped).
 
  • #4
Coelum said:
If ## E=V_0 ## the Schrodinger equation outside the well becomes
I'm not sure about setting ##E = V_0## at this stage. It should result in no solutions,
Coelum said:
$$ -\frac{\hbar^2}{2m}\frac{d^2\Psi}{dx^2} = 0 $$
with solution a linear function (## A,B\in \mathcal R ##)
$$ \Psi(x)=Ax+B. $$
Inside the well the S.E. is
$$ -\frac{\hbar^2}{2m}\frac{d^2\Psi}{dx^2} = E\Psi $$
$$ \frac{d^2\Psi}{dx^2} = -\frac{2mE}{\hbar^2}\Psi = -K^2\Psi $$
with solution (## C,D\in \mathcal R ##)
$$ \Psi(x)=C\sin(Kx)+D\cos(Kx). $$
The normalization condition for a single particle imposes ## A=B=0 ## on both sides of the well. The boundary conditions for the continuity of ## \Psi ## and its first derivative at the borders of the well lead to the null solution as the only solution. That's what I meant in my previous statements.
That's not the approach I understood in your OP. If you follow through with the mathematics here, then you end up with no solutions (for numerical reasons) with ##E = V_0##.
Coelum said:
If you still believe my reasoning is wrong, please help me by pointing out the error in the derivation above (including computing the boundary conditions, whose boring details I shamelessly skipped).
There's nothing wrong if you try to solve the equation fully where ##E = V_0## and show there are no solutions.
 
  • #5
PeroK said:
I'm not sure about setting ##E = V_0## at this stage. It should result in no solutions,

That's not the approach I understood in your OP. If you follow through with the mathematics here, then you end up with no solutions (for numerical reasons) with ##E = V_0##.

There's nothing wrong if you try to solve the equation fully where ##E = V_0## and show there are no solutions.
Great, I think we agree now. Thanks again.
 
  • #6
From what I know for scattering, where the problem is slightly different as the spatial coordinate is then radial, ##r \in [0,\infty)##, I expect the following.

The case ##E=V_0## would correspond to a scattering (unbounded) state, increasing linearly outside the well, corresponding to an infinite wavelength. That seems to be what you found. Inside the well, the wave function will be oscillating (the amount of oscillation increasing with the number of bound states).
Coelum said:
The normalization condition for a single particle imposes A=B=0 on both sides of the well.
I think this is where it goes wrong. Scattering (continuum) wave functions are not square integrable, but there are normalization convention one can use. Of course, these states cannot correspond to actual physical states, but are useful for calculating things (just like plane waves).
 
  • #7
DrClaude said:
From what I know for scattering, where the problem is slightly different as the spatial coordinate is then radial, ##r \in [0,\infty)##, I expect the following.

The case ##E=V_0## would correspond to a scattering (unbounded) state, increasing linearly outside the well, corresponding to an infinite wavelength. That seems to be what you found. Inside the well, the wave function will be oscillating (the amount of oscillation increasing with the number of bound states).

I think this is where it goes wrong. Scattering (continuum) wave functions are not square integrable, but there are normalization convention one can use. Of course, these states cannot correspond to actual physical states, but are useful for calculating things (just like plane waves).
I agree. In fact, I just worked out a non-normalizable solution. I started asking myself: can the energy of a particle in a free state be equal to V0?

In order to simplify the treatment, I assume we have a stationary flow of particles. Then, since we do not require normalization, a solution to the stationary Schrodinger equation exists. It is a real constant outside the well and a sum of sine and cosine inside (details depend on the selection of the origin). More interestingly, we get the following condition: L*sqrt(2mV_0)/h must be an integer number. Otherwise, the wave-function is zero.

My question is about the interpretation of the solution. For E>V_0, the solution of the problem is a sum of two complex exponentials representing flows in opposite direction of the x axis. Can the same interpretation be applied to our case? Furthermore, it looks like this solution has no classical equivalent. Is that correct?
 
  • Like
Likes PeroK
  • #8
DrClaude said:
The case ##E=V_0## would correspond to a scattering (unbounded) state, increasing linearly outside the well, corresponding to an infinite wavelength.
Are you sure this isn't a degenerate solution? Usually the non-normalisable wave-functions are combined in a wave-packet, with (I assume) the ##E## as the most probable energy. Here, ##E = V_0## is on the lower limit of possible scattering energies, so it ought to have a probability density of zero in any wave-packet.
 
  • #9
Coelum said:
More interestingly, we get the following condition: L*sqrt(2mV_0)/h must be an integer number. Otherwise, the wave-function is zero.
I think you are requiring the potential wave to a have quasi-bound state at ##E = V_0##, which is not necessary. Solutions exist for any value of ##L## and ##V_0##.
Coelum said:
My question is about the interpretation of the solution. For E>V_0, the solution of the problem is a sum of two complex exponentials representing flows in opposite direction of the x axis. Can the same interpretation be applied to our case? Furthermore, it looks like this solution has no classical equivalent. Is that correct?
It a scattering state corresponding to zero momentum, so particles not moving/scattering. What this means exactly I can't say.
 
  • #10
PeroK said:
Are you sure this isn't a degenerate solution? Usually the non-normalisable wave-functions are combined in a wave-packet, with (I assume) the ##E## as the most probable energy. Here, ##E = V_0## is on the lower limit of possible scattering energies, so it ought to have a probability density of zero in any wave-packet.
As I said above, I don't know how to physically interpret zero-energy scattering. It won't appear in any realistic condition, but it is still part of the set of solutions to the problem, and it used for instance to calculate scattering lengths, as shown in this figure from the Sakurai textbook:
 

Attachments

  • scattering.jpg
    scattering.jpg
    16.5 KB · Views: 57
  • Informative
Likes PeroK
  • #11
Thanks to both PeroK and DrClaude for the very interesting discussion. Now I understand much better what I found.
 
  • Like
Likes PeroK
  • #12
DrClaude said:
As I said above, I don't know how to physically interpret zero-energy scattering
I don't think anybody does.

Note that Δp = 0 so Δx must be infinite. I don't know how one would prepare such a state, but it would be hard to describe it as "bound".
 
  • #13
Vanadium 50 said:
I don't think anybody does.

Note that Δp = 0 so Δx must be infinite. I don't know how one would prepare such a state, but it would be hard to describe it as "bound".
Yes, it looks to me like a mix of bound and unbound states. On one hand, the classical corresponding state is bound. On the other hand, it is not normalizable, so it should represent a stream of particles. Also, there is a well-defined condition for it to exist. Not all wells allow such a state. Overall, really strange.
 

1. What is a finite well problem?

A finite well problem is a mathematical model used in quantum mechanics to describe the behavior of a particle confined to a finite region with a potential barrier. It is commonly used to study the energy levels and wave functions of particles in a confined space.

2. How is the finite well problem solved?

The finite well problem is typically solved using the Schrödinger equation, which is a mathematical equation that describes the time evolution of a quantum system. The solutions to this equation provide information about the energy levels and wave functions of the particle in the finite well.

3. What is V0 in the finite well problem?

V0 is the potential energy of the barrier in the finite well. It represents the height of the barrier that the particle must overcome to move from one side of the well to the other. In some cases, V0 can also represent the depth of the well, depending on the specific problem being studied.

4. Can the energy of the particle be equal to V0 in the finite well problem?

Yes, it is possible for the energy of the particle to be equal to V0 in the finite well problem. This occurs when the particle has enough energy to overcome the potential barrier and escape the well. In this case, the particle is considered to be in a state of unbound energy.

5. What are the implications of the energy being equal to V0 in the finite well problem?

If the energy of the particle is equal to V0, it means that the particle has enough energy to escape the well and is no longer confined to the finite region. This has important implications for the behavior and properties of the particle, as it is no longer subject to the potential barrier and can move freely in space.

Similar threads

  • Quantum Physics
Replies
11
Views
2K
  • Quantum Physics
Replies
10
Views
1K
  • Quantum Physics
Replies
21
Views
400
Replies
2
Views
700
Replies
2
Views
700
  • Quantum Physics
Replies
1
Views
1K
  • Introductory Physics Homework Help
Replies
2
Views
475
  • Quantum Physics
Replies
3
Views
2K
  • Quantum Physics
Replies
1
Views
746
Replies
13
Views
1K
Back
Top